1
$\begingroup$

Let $f(n)$ be a real-valued sequence defined for $n \in \mathbb{N}$, with $f(n) > 0$ for all $n$. Define a new sequence: $$ g(n) = \log_b(f(n)) $$

I know that when $0 < b < 1$, the logarithmic function is strictly decreasing. Thus, if $x_1 < x_2$, then $$ \log_{1/2}(x_1) > \log_{1/2}(x_2) $$

Logically this should means that when $f(n)$ attains a minimum (maximum), then $g(n)$ attains a maximum (minimum)?

After if I have a $b>1$, if $f(n)$ have a minimum (maximum) then $g(n)$ have a maximum (minimum)?

For example: I want found the infimum and supremum of the following set, specifying whether they correspond to a maximum and/or minimum: $$ X = \left\{ \log_{\frac{1}{2}}(n^2 - 7n + 13) : n \in \mathbb{N} \right\} $$ I observe that the argument of the logarithm is the parabola: $$ f(n) = n^2 - 7n + 13, $$ which opens upwards (the coefficient of $n^2$ is positive), thus it has an absolute minimum at its vertex. Let's calculate it: $$ n_V = \frac{7}{2} = 3.5, \quad f(3.5) = (3.5)^2 - 7 \cdot 3.5 + 13 = 12.25 - 24.5 + 13 = 0.75. $$ Since $n \in \mathbb{N}$, consider the nearest integers: $$ f(3) = 9 - 21 + 13 = 1, \quad f(4) = 16 - 28 + 13 = 1. $$ Therefore, the minimum value taken by the parabola for $n \in \mathbb{N}$ is 1, reached at $n=3$ and $n=4$. Now consider the function: $$ g(n) = \log_{\frac{1}{2}}(f(n)) = \log_{\frac{1}{2}}(n^2 - 7n + 13). $$ Since the base of the logarithm is in the interval $]0,1[$, the logarithmic function is strictly decreasing. Hence, if $x_1 < x_2$ then $$ \log_{\frac{1}{2}}(x_1) > \log_{\frac{1}{2}}(x_2). $$

This means that when $f(n)$ is minimum, $g(n)$ is maximum?

$$ \begin{array}{c|c|c} n & f(n) = n^2 - 7n + 13 & g(n) = \log_{\frac{1}{2}}(f(n)) \\ \hline 1 & 7 & \approx -2.807 \\ 2 & 3 & \approx -1.585 \\ 3 & 1 & 0 \\ 4 & 1 & 0 \\ 5 & 3 & \approx -1.585 \\ 6 & 7 & \approx -2.807 \\ 7 & 13 & \approx -3.700 \\ 8 & 21 & \approx -4.393 \\ 9 & 31 & \approx -4.954 \\ 10 & 43 & \approx -5.426 \\ \end{array} $$

The supremum of $X$ is $\sup X = \max X = 0$, corresponding to the maximum value reached at $n=3$ and $n=4$.
The infimum of $X$ is $\inf X = -\infty$, because $f(n) \to +\infty$ as $n \to +\infty$ and thus $g(n) = \log_{\frac{1}{2}}(f(n)) \to -\infty$. There is no minimum because the infimum $-\infty$ is not attained by any element of the set.

Same question for the exponential function $$g(n) = b^{f(n)}.$$ for the exercise:

$$X=\left\{\left(\frac12\right)^{n^2-7n+4} \mid n\in \mathbb N\right\}$$

looking the exponent of $1/2$.

I have create table $$ \begin{array}{|c|c|c|c|} \hline \textbf{Function} & \textbf{Base interval} & \textbf{Monotonicity} & \textbf{Effect on min/max of } f(n) \\ \hline g(n)=\log_b(f(n)) & 0 < b < 1 & \text{decreasing } f & \text{min of } f \rightarrow \text{max of } g \\ g(n)=\log_b(f(n)) & 0 < b < 1 & \text{increasing } f& \text{max of } f \rightarrow \text{min of } g \\ g(n)=\log_b(f(n)) & b > 1 & \text{decreasing } f& \text{min of } f \rightarrow \text{min of } g \\ g(n)=\log_b(f(n)) & b > 1 & \text{increasing } f& \text{max of } f \rightarrow \text{max of } g \\ b^{f(n)} & 0 < b < 1 & \text{decreasing } f & \text{min of } f \rightarrow \text{max of } g \\ b^{f(n)} & 0 < b < 1 & \text{increasing } f& \text{max of } f \rightarrow \text{min of } g \\ b^{f(n)} & b > 1 & \text{decreasing } f& \text{min of } f \rightarrow \text{min of } g\\ b^{f(n)} & b > 1 & \text{increasing } f& \text{max of } f \rightarrow \text{max of } g \\ \hline \end{array} $$

to understand whether there exist theorems or proofs that logically establish the correctness of the assertions about how the monotonicity of $f$ affects $g$, without relying on derivatives, and if such theorems are available.

This concept for me is important to found the $\inf X (\min X)$, $\sup X (\max X)$ of a generic set $X$ that involves with exponentials and logarithms.

$\endgroup$
6
  • $\begingroup$ What do you mean by "min of $f$" if $f$ is monotonically decreasing? Why would it have a minimum? $\endgroup$ Commented Oct 11 at 22:36
  • $\begingroup$ Are you here referring to "supremum"/"infimum", rather than "maximum"/"minimum" $\endgroup$ Commented Oct 12 at 3:58
  • 1
    $\begingroup$ @JCQ Hi, please, could you explain that better? Are you referring to my table? $\endgroup$ Commented Oct 12 at 9:12
  • $\begingroup$ Almost everywhere including the table, actually. As @Sammy Black has said, if $f(n)$ is strictly decreasing, it cannot has a minimum, but it can has an infimum. For example $\displaystyle f(n)=\frac{1}{n}$ is strictly decreasing without a minimum, and its infimum is $0$. The same works for maximum/supremum. $\endgroup$ Commented Oct 12 at 9:18
  • $\begingroup$ @JCQ Ah ok. Can I ask a courtesy, please? Can you edit my table referring to the comments? Thank you. $\endgroup$ Commented Oct 12 at 9:19

0

You must log in to answer this question.

Start asking to get answers

Find the answer to your question by asking.

Ask question

Explore related questions

See similar questions with these tags.